[Linear Algebra] - Resolving a system

Click For Summary
The discussion revolves around a problem involving three truck drivers and their purchases at a cafe, leading to a system of equations. The equations derived from their purchases are 4s + 1c + 10d = 8.45 and 3s + 1c + 7d = 6.3, with the goal of determining the cost of a sandwich, coffee, and doughnut for the third driver. Participants note that the system has infinite solutions due to having four variables and only three equations, suggesting that 'a' is not a variable but rather a representation of the third driver's total. The objective is to express the costs of the items in terms of 'a'. The conversation highlights the need for clarity in defining variables and the structure of the equations.
Tosh5457
Messages
130
Reaction score
28

Homework Statement



Three truck drivers went into a roadside cafe. One truck driver purchased four sandwiches, a cup of coffee and ten doughnuts for $8.45. Another drivers purchased three sandwiches, a cup of coffee and seven doughnuts for $6.3. What did the third truck driver pay for a sandwich, a cup of coffee and a doughnut?

Homework Equations


The Attempt at a Solution



The system's equations are:
4s + 1c + 10d = 8.45
3s + 1c + 7d = 6.3
1s + 1c + 1d = a

The system has infinite solutions, right? There are 4 variables for 3 equations. I can't determine a.
 
Last edited:
Physics news on Phys.org
I don't think 'a' is intended to be a variable. I believe the objective is to solve for x,y, z in terms of 'a'. Thus, 3 equations and 3 unknowns.
 
hotvette said:
I don't think 'a' is intended to be a variable. I believe the objective is to solve for x,y, z in terms of 'a'. Thus, 3 equations and 3 unknowns.

I wrote the problem wrongly on the first time, I just edited it.
 
Question: A clock's minute hand has length 4 and its hour hand has length 3. What is the distance between the tips at the moment when it is increasing most rapidly?(Putnam Exam Question) Answer: Making assumption that both the hands moves at constant angular velocities, the answer is ## \sqrt{7} .## But don't you think this assumption is somewhat doubtful and wrong?

Similar threads

  • · Replies 13 ·
Replies
13
Views
2K
  • · Replies 6 ·
Replies
6
Views
2K
  • · Replies 21 ·
Replies
21
Views
3K
  • · Replies 4 ·
Replies
4
Views
2K
  • · Replies 1 ·
Replies
1
Views
6K
  • · Replies 3 ·
Replies
3
Views
4K
Replies
4
Views
6K
Replies
2
Views
3K
  • · Replies 4 ·
Replies
4
Views
3K
Replies
4
Views
2K